At a café,
3 teas and 1 coffee cost £5.10
1 tea and 4 coffees cost £8.30
Work out the cost of 1 tea and the cost of 1 coffee.

Answers

Answer 1

As a result, one cup of tea costs £0.65 and one cup of coffee costs £3.15.

Why do we determine costs?

Cost computation helps in deciding on pricing, manufacturing output, and sales. It also helps in figuring out the costs of the products and services the company sells.

Let's assume that a cup of tea costs t and a cup of coffee costs c.

We can infer the following based on the initial piece of knowledge:

3t + 1c = 5.10 --------------(1)

We learn the following from of the second piece of information:

1t + 4c = 8.30 --------------(2)

We can find the solutions to t and c because we have two equations with two variables.

Equation (1) is multiplied by 4 and equation (2) is taken away to yield the following result:

9t = 5.90

Therefore:

t = 0.65

Using t = 0.65 as the replacement in equation (1), we get:

3(0.65) + 1c = 5.10

1c = 5.10 - 1.95

1c = 3.15

Therefore:

c = 3.15

As a result, one cup of tea costs £0.65 and one cup of coffee costs £3.15.

To know more about cost visit:

https://brainly.com/question/25109150

#SPJ1


Related Questions

The terminal ray of angle A, drawn in standard position, passes through the point (-4,
-6). What is the value of sec(A)?
Give your answer in simpliest radical form.

Answers

The value of sec A as required to be determined in the task content is; -√13 / 2.

What value represents sec A in the given scenario?

As evident from the task content; it follows that the terminal ray of angle A, drawn in standard position, passes through the point (-4, -6).

Therefore, the length that the line from the origin to A has length;

L = √((-4)² + (-6)²)

L = √52.

On this note, it follows that the value of sec A which is represented by; hypothenuse/ adjacent is;

sec (A) = -√52 / 4

sec (A) = -√13 / 2.

Ultimately, the value of sec (A) as required is; -√13 / 2.

Read more on trigonometric ratios;

https://brainly.com/question/10417664

#SPJ1

the radius of a right circular cone is increasing at a rate of 1.4 in/s while its height is decreasing at a rate of 2.6 in/s. at what rate is the volume of the cone changing when the radius is 144 in. and the height is 138 in.?

Answers

Answer:

Let's use the formula for the volume of a right circular cone to solve this problem:

V = (1/3)πr^2h

We are given that the radius is increasing at a rate of 1.4 in/s and the height is decreasing at a rate of 2.6 in/s. We want to find the rate at which the volume is changing when the radius is 144 in. and the height is 138 in. In other words, we want to find dV/dt when r = 144 and h = 138.

Using the chain rule of differentiation, we can express the rate of change of the volume as follows:

dV/dt = (dV/dr) (dr/dt) + (dV/dh) (dh/dt)

To find dV/dr and dV/dh, we differentiate the formula for the volume with respect to r and h, respectively:

dV/dr = (2/3)πrh

dV/dh = (1/3)πr^2

Substituting the given values and their rates of change, we have:

dV/dt = (2/3)π(144)(138)(1.4) + (1/3)π(144)^2(-2.6)

dV/dt = 55,742.4 - 1,994,598.4

dV/dt = -1,938,856 in^3/s

Therefore, when the radius is 144 in. and the height is 138 in., the volume of the cone is decreasing at a rate of approximately 1,938,856 cubic inches per second.

Step-by-step explanation:

PLSS HELP IVE TRIED EVERYTHING

Answers

Answer:

Step-by-step explanation: To obtain the function g(x) that represents the indicated transformations of the graph of f(x) = 2, which are a translation 1 unit up followed by a horizontal stretch by a factor of 2, we can follow these steps:

To translate f(x) = 2 one unit up, we can add 1 to the function: f(x) + 1.

To horizontally stretch f(x) + 1 by a factor of 2, we can multiply the input (x) by 1/2: f(1/2 x) + 1.

Therefore, the function g(x) that represents the indicated transformations of f(x) is:

g(x) = f(1/2 x) + 1

g(x) = 2(1/2 x) + 1

g(x) = x + 1

a jar contains 6 red marbles numbered 1 to 6 and 12 blue marbles numbered 1 to 12. a marble is drawn at random from the jar. find the probability of the given event. write your answers as reduced fractions. (a) the marble is red your answer is : (b) the marble is odd-numbered

Answers

a) The total number of marbles is 6 + 12 = 18.P(a red marble) = 6/18 = 1/3

b) There are 12 odd-numbered marbles in total. P(an odd-numbered marble) = 12/18 = 2/3

The probability that the marble drawn from the jar is red can be found using the formula for probability. The probability formula can be written as the ratio of the number of favorable outcomes to the total number of possible outcomes. P(a red marble) = number of red marbles / total number of marbles. In this case, there are 6 red marbles and 12 blue marbles in the jar. Therefore, the total number of marbles is 6 + 12 = 18.P(a red marble) = 6/18 = 1/3(b) The probability that the marble drawn from the jar is odd-numbered can be found using the formula for probability. The probability formula can be written as the ratio of the number of favorable outcomes to the total number of possible outcomes. P(an odd-numbered marble) = number of odd-numbered marbles / total number of marbles. In this case, there are 6 red marbles numbered 1 to 6 and 12 blue marbles numbered 1 to 12 in the jar. Therefore, the total number of marbles is 6 + 12 = 18.To find the number of odd-numbered marbles, we need to count the number of red and blue marbles numbered 1, 3, 5, 7, 9, 11. There are 6 odd-numbered red marbles and 6 odd-numbered blue marbles. Therefore, there are 12 odd-numbered marbles in total. P(an odd-numbered marble) = 12/18 = 2/3

Learn more about Probability

brainly.com/question/30034780

#SPJ11

What is the value of 3x + 6 if x = -5

Answers

Answer:

-9

Step-by-step explanation:

x = -5

3x + 6

Since x = -5..

Do this

3(-5) + 6

Perform

-15 + 6

Answer: -9

Therefore, when x is equal to -5, the value of 3x + 6 is -9.

What is equation?

An equation is a statement that expresses the equality of two mathematical expressions using mathematical symbols such as variables, numbers, and mathematical operations. The equality is represented by an equal sign "=" between the two expressions. Equations are used to represent mathematical relationships and solve problems in various fields such as physics, chemistry, engineering, and economics.

Given by the question.

3x + 6 = 3(-5) + 6

= -15 + 6

= -9

To learn more about equality:

https://brainly.com/question/12323068

#SPJ1

Where is point c if c 2 units closer to b than it is a?

Answers

As per the points A, B, and C are colinear points and the point C lies between points A and B on the line.

Let's begin by finding the distance between points A and B. Using the distance formula equation, we can substitute the values of the x and y coordinates of A and B:

AB = √((0 - 5)² + (5 - (-5))²)

= √(25 + 100)

= √125

= 5√5

Therefore, the distance between points A and B is 5√5.

Similarly, we can find the distance between points B and C:

BC = √((2 - 0)² + (1 - 5)²)

= √4 + 16

= √20

= 2√5

Finally, we can find the distance between points A and C:

AC = √((2 - 5)² + (1 - (-5))²)

= √9 + 36

= √45

= 3√5

Alternatively, we can use the equation of the line passing through any two of these points and check if the third point lies on that line.

Let's use the points A and B to find the equation of the line passing through them:

y - (-5) = ((5 - (-5)) / (0 - 5))(x - 5)

y + 5 = (10 / (-5))(x - 5)

y + 5 = -2(x - 5)

y + 5 = -2x + 10

y = -2x + 5

Now, let's check if point C lies on this line by substituting its coordinates into the equation:

1 = -2(2) + 5

1 = 1

Since the equation is true, we can conclude that points A, B, and C are collinear. Moreover, since point C lies between points A and B on the line, we can say that C lies on segment AB.

To know more about colinear here

https://brainly.com/question/24244967

#SPJ4

Complete Question:

Given points A, B, and C. Find AB, BC, and AC. Are A, B, and C collinear? If so, which point lies between the other two? A(5, −5), B(0,5), C(2, 1)

a credit risk study found that an individual with good credit score has an average debt of $15,000. if the debt of an individual with good credit score is normally distributed with standard deviation $3,000, determine the shortest interval that contains 95% of the debt values.

Answers

The shortest interval that contains 95% of the debt values is $9,492.02 to $20,507.98

How do we calculate the interval values?

Given that a credit risk study found that an individual with good credit score has an average debt of $15,000 and the debt of an individual with good credit score is normally distributed with standard deviation $3,000.

Then the 95% confidence interval can be calculated as follows:

Upper limit: µ + Zσ

Lower limit: µ - Zσ

Where

µ is the mean ($15,000)Z is the z-scoreσ is the standard deviation ($3,000).

The z-score corresponding to a 95% confidence interval can be found using the standard normal distribution table.

The area to the left of the z-score is 0.4750 and the area to the right is also 0.4750.

The z-score corresponding to 0.4750 can be found using the standard normal distribution table as follows:z = 1.96Therefore

Upper limit: µ + Zσ= $15,000 + 1.96($3,000) = $20,880

Lower limit: µ - Zσ= $15,000 - 1.96($3,000) = $9,120.02

The shortest interval that contains 95% of the debt values is $9,492.02 to $20,507.98.

See more about confidence interval at: https://brainly.com/question/15712887

#SPJ11

Write 2/7 + 1/4 as a sum of two equivalent fractions with the same denominator

Answers

2/7 + 1/4 = 15/28 ≅ 0.5357143


Add: 2/7 + 1/4 = 2 · 4/7 · 4 + 1 · 7/4 · 7 = 8/28 + 7/28 = 8 + 7/28 = 15/28 

It is suitable to adjust both fractions to a common (equal, identical) denominator for adding, subtracting, and comparing fractions. The common denominator you can calculate as the least common multiple of both denominators - LCM(7, 4) = 28. It is enough to find the common denominator (not necessarily the lowest) by multiplying the denominators: 7 × 4 = 28. In the following intermediate step, it cannot further simplify the fraction result by canceling.In other words - two sevenths plus one quarter is fifteen twenty-eighths.

The denominator will be 28
But for the total it would be 15/28

Jenny works at Sammy's Restaurant and is paid according to the rates in the following
table.

Jenny's weekly wage agreement
Basic wage $600.00
PLUS
$0.90 for each customer served.

In a week when Jenny serves n customers, her weekly wage, W, in dollars, is given by
the formula
W = 600+ 0.90n.

(i) Determine Jenny's weekly wage if she served 230 customers.

(ii) In a good week, Jenny's wage is $1 000.00 or more. What is the LEAST number
of customers that Jenny must serve in order to have a good week?

(iii) At the same restaurant, Shawna is paid a weekly wage of $270.00 plus $1.50 for
each customer she serves.
If W, is Shawna's weekly wage, in dollars, write a formula for calculating Shawna's
weekly wage when she serves m customers.

(iv) In a certain week, Jenny and Shawna received the same wage for serving the same
number of customers.
How many customers did they EACH serve?

Answers

Jenny works at Sammy's Restaurant and is paid according to the rates in the following

table.

Jenny's weekly wage agreement

Basic wage $600.00

PLUS

$0.90 for each customer served.

In a week when Jenny serves n customers, her weekly wage, W, in dollars, is given by

the formula

W = 600+ 0.90n.

(i) Determine Jenny's weekly wage if she served 230 customers.

(ii) In a good week, Jenny's wage is $1 000.00 or more. What is the LEAST number

of customers that Jenny must serve in order to have a good week?

(iii) At the same restaurant, Shawna is paid a weekly wage of $270.00 plus $1.50 for

each customer she serves.

If W, is Shawna's weekly wage, in dollars, write a formula for calculating Shawna's

weekly wage when she serves m customers.

(iv) In a certain week, Jenny and Shawna received the same wage for serving the same

number of customers.

How many customers did they EACH serve?

3 / 3

(i) If Jenny serves 230 customers, her weekly wage is

W = 600 + 0.90n = 600 + 0.90(230) = $807.00

Therefore, Jenny's weekly wage if she serves 230 customers is $807.00.

(ii) We want to find the least number of customers, n, that Jenny must serve in order to earn $1,000 or more. That is,

600 + 0.90n ≥ 1,000

0.90n ≥ 400

n ≥ 444.44

Since n must be a whole number, Jenny must serve at least 445 customers in order to earn $1,000 or more in a week.

(iii) Shawna's weekly wage, W, in dollars, when she serves m customers is given by the formula:

W = 270 + 1.50m

Therefore, Shawna's weekly wage when she serves m customers is $270.00 plus $1.50 for each customer she serves.

(iv) Let's assume that Jenny and Shawna received the same wage, W, for serving the same number of customers, x. Then we have:

Jenny's wage = 600 + 0.90x

Shawna's wage = 270 + 1.50x

Setting these two expressions equal to each other, we get:

600 + 0.90x = 270 + 1.50x

330 = 0.60x

x = 550

Therefore, Jenny and Shawna each served 550 customers.

Translate the sentence into an equation.
Eight times the sum of a number and 5 equals 7 .

Answers

Answer:

Let the number be represented by the variable "x". Then, the sentence can be translated into the following equation:

8(x + 5) = 7

This prism has a right triangle for a base. The volume of the prism is
54 cubic units. What is the value of h?
A
a
CA
6.
OF

mi
E

Answers

Answer:

9

Step-by-step explanation:

The base of the prism is a right triangle.

The hypotenuse measures 5. One leg measures 4.

The other leg must measure 3 since it is the Pythagorean triple 3, 4, 5.

Area of the base = 3 × 4 / 2 = 6

V = Bh

54 = 6 × h

h = 9

if you weigh 160 pounds, how many drinks in four hours would you need to drink to be definitely illegal?

Answers

According to the provided scenario, if you weigh 160 pounds, then 3 drinks in four hours would make you definitely illegal.

If a person weighs 160 pounds and drinks alcohol at a moderate rate, then after 3 drinks in four hours, their BAC (blood alcohol concentration) would be around 0.08, which is considered legally impaired and definitely illegal. However, it is important to note that this estimate is based on various factors such as the person's gender, age, and metabolism, and can vary from person to person.

Therefore, it is always advisable to drink responsibly and not drive after consuming alcohol.

To learn more about drinking refer :

https://brainly.com/question/10922020

#SPJ11

You are playing a game with a friend. It costs you $2 to play. If you roll a 1 on a 6-sided die you win $4. If you roll a 2, 3, 4, 5, or 6 you win nothing and lose $2 the cost to play. How much should the player be willing to pay to play this game and not lose money in the long run?

Answers

The player should be willing to pay up to $1.33 to play this game and not lose money in the long run.

The expected value is the sum of the products of each possible outcome and its probability. Let's calculate the expected value of the game:

E(X) = (1/6) * $4 + (5/6) * (-$2)

E(X) = $0.67

This means that on average, the player can expect to win $0.67 per game. Since it costs $2 to play, the player should not be willing to pay more than $2 - $0.67 = $1.33 to play the game and not lose money in the long run.

Probability theory is based on axioms, which are basic assumptions about the nature of probability. It is used to quantify uncertainty and to make predictions based on the available information. Probability is expressed as a number between 0 and 1, with 0 meaning an event is impossible, and 1 meaning an event is certain.

The concept of probability is used in a variety of fields, including statistics, economics, engineering, and physics. In statistics, probability is used to model random variables, estimate parameters, and test hypotheses. In economics, probability is used to model financial risks and decision-making under uncertainty. In engineering and physics, probability is used to model complex systems and predict the behavior of particles.

To learn more about Probability visit here:

brainly.com/question/30034780

#SPJ4

Two ropes are attached to a tree, and forces of F_1 = 1.31 + 4.6J n and F_2 = 3.2i + 6.8j n are applied. The forces are coplanar (in the same plane). What is the resultant (net force) of these two force vectors (in N)? (Express your answer in vector form.) Find the magnitude (in N) and direction (in degrees counterclockwise from the +x-axis) of this net force.

Answers

The magnitude and direction of the net force are found by adding the two forces together as resultant force vectors.

a) 11.82 N

b) 74.07°

To find the net force, we add the two force vectors F_1 and F_2:

Fnet = F_1 + F_2

Fnet = (1.31 + 4.6j) N + (3.2i + 6.8j) N

Fnet = 3.2i + (1.31 + 4.6j + 6.8j) N

Fnet = 3.2i + (1.31 + 11.4j) N

To find the magnitude of the net force, we use the Pythagorean theorem:

|Fnet| = sqrt[(3.2)^2 + (1.31 + 11.4)^2] N

|Fnet| ≈ 11.6 N

To find the direction of the net force, we use the inverse tangent function:

θ = tan^(-1)(y/x)

θ = tan^(-1)(11.4/3.2)

θ ≈ 73.8 degrees

Since the net force is in the first quadrant, the direction counterclockwise from the +x-axis is simply θ:

Direction = 73.8 degrees counterclockwise from the +x-axis

Therefore, the net force is Fnet = 3.2i + (1.31 + 11.4j) N, with a magnitude of approximately 11.6 N and a direction of approximately 73.8 degrees counterclockwise from the +x-axis.

To know more about the magnitude and direction of the "net force": https://brainly.com/question/10409611

#SPJ11

Smallest possible answer.

Answers

The smallest possible values for "a" and "b" that satisfy these conditions are when a = 1 and b = 3. This is because 1 is the smallest factor of 15, and 3 is the smallest multiple of 3.

What is integers?

The Latin term "Integer," which implies entire or intact, is where the word "integer" first appeared. Zero, positive numbers, and negative numbers make up the particular set of numbers known as integers.

Let's call the two unknown numbers as "a" and "b"

Since "a" is a factor of 15, the possible values for "a" are 1, 3, 5, and 15.

Since "b" is a multiple of 3, the possible values for "b" are 3, 6, 9, 12, 15, and so on.

The smallest possible values for "a" and "b" that satisfy these conditions are when a = 1 and b = 3. This is because 1 is the smallest factor of 15, and 3 is the smallest multiple of 3.

Learn more about integers on:

https://brainly.com/question/12399107

#SPJ1

If A={1,2,3}, B= {} show that A is not equal to B

Answers

In set theory, two sets are considered equal if they have the same elements. In this case, A is a set containing the elements 1, 2, and 3, while B is an empty set (also known as the null set),

A contains three distinct elements, and B contains none, we can conclude that A and B are not equal, i.e., A is not equal to B.

A ≠ B

Set theory is a branch of mathematics that studies collections of objects, called sets, and the relationships between them. A set is defined as a well-defined collection of distinct objects, which can be anything from numbers and letters to more abstract concepts like functions and geometrical shapes. The set theory provides a foundation for other areas of mathematics, including algebra, topology, and logic.

One of the fundamental concepts of set theory is the notion of membership, which states that an object either belongs to a set or does not. Sets can also be combined through operations such as union, intersection, and complementation, and the relationships between sets can be represented using Venn diagrams.

To learn more about Set theory visit here:

brainly.com/question/30764677

#SPJ4

choice matrix is shown. Complete the choice matrix by selecting the value equivalent to each function output. 

Consider the functions shown.

f(x) = -3(2^x)

g(x) = -3 + 2x

h(x) = 2(3^x)

j(x) = -3 – 2x


Answers

Answer:

f(x) = -3(2^x)

g(x) = -3 + 2x

h(x) = 2(3^x)

j(x) = -3 – 2x

So,

f(2) = -3(2^2)

f(2) = -3(4)

f(2) = -12

g(-2) = -3 + 2(-2)

g(-2) = -3 -4

g(-2) = -7

h(2) = 2(3^2)

h(2) = 2(9)

h(2) = 18

j(-2) = -3 – 2(-2)

j(-2) = -3 –4

j(-2) = -7

It is known that diskettes produced by a certain company will be defective with probability 0.01, independently of each other. The company sells the diskettes in packages of size 10 and offers a money-back guarantee that at most 1 of the 10 diskettes in the package will be defective.If someone buys 3 packages, what is the probability that he or she will return exactly 1 of 3 packages?

Answers

The probability of someone returning exactly 1 of the 3 packages can be calculated as:P(1 out of 3 packages is returned) = C(3, 1) × P(0 or 1 diskette is defective)¹ × (1 - P(0 or 1 diskette is defective))²P(1 out of 3 packages is returned) = C(3, 1) × (0.9043820371)¹ × (0.0956179629)²P(1 out of 3 packages is returned) = 0.2448700124Therefore, the required probability of someone returning exactly 1 of the 3 packages is 0.2448700124.

The given data from the question is that the company produces diskettes which have the probability of being defective as 0.01. The packages that are sold have a size of 10 and the guarantee says that there can be at most one defective diskette in the package. Now, the question is to find the probability of someone returning exactly 1 of the 3 packages that they have bought.  So, the given data can be summarized as:Given:Probability of the diskette being defective, p = 0.01Guarantee: At most one diskette in the package of size 10 is defective.Now, let's solve the problem using probability theory

Probability of 1 diskette being defective in a package of size 10 can be calculated as:P(defective) = p = 0.01P(non-defective) = 1 - p = 0.99Using the given guarantee, probability of at most one defective diskette in a package of size 10:P(0 or 1 diskette is defective) = P(0 defective) + P(1 defective)P(0 or 1 diskette is defective) = C(10, 0) × (0.99)¹⁰ + C(10, 1) × (0.99)⁹ × (0.01)P(0 or 1 diskette is defective) = 0.9043820371Using the above probability

Learn more about Probability

brainly.com/question/30034780

#SPJ11

formation about a sample is given. Assume that the sampling distribution is symmetric and bell-shaped. P_1 - P_2 = 0.15 and the margin of error for 95% confidence is 5%. (a) Indicate the parameter being estimated.(b) Use the information to give a 95% confidence interval.

Answers

(a) Parameter being estimated in the given information is the difference between two proportions (p_1 - p_2).

(b) A 95% confidence interval is given by  (0.075, 0.225)

(a) The parameter being estimated is the difference between two population proportions, which is denoted by (p_1 - p_2).

(b) The margin of error for a 95% confidence interval is 5%, which means that the critical value of z is 1.96 (obtained from a standard normal distribution table). Using the formula for the margin of error, we can write:

1.96 * √(p_1_hat*(1-p_1_hat)/n_1 + p_2_hat*(1-p_2_hat)/n_2) = 0.05

where p_1_hat and p_2_hat are the sample proportions from the two samples, and n1 and n2 are the sample sizes.

Solving for p_1_hat - p_2_hat, we get:

p1_hat - p2_hat = ±0.075

Since we are interested in a 95% confidence interval, we can subtract and add this value from P1 - P2 to obtain the interval:

P_1 - P_2 ± 0.075

Substituting the given value of P_1 - P_2 = 0.15, we get:

95% Confidence Interval: (0.075, 0.225)

To know more about the "confidence interval" concept: https://brainly.com/question/17097944

#SPJ11

7) Roy buys pizza for his friends. A whole pizza costs P 190. 00 and P 40. 00 for every
additional topping. If he spent P 1070 for pizza with 3 sets of additional toppings, how
many whole pizzas did he buy?

Answers

Roy purchased whole pizzas for P 190.00 each. To determine the number of whole pizzas he bought, we can divide the total cost of the pizzas by the cost of each pizza. Therefore, the calculation P 950.00 / P 190.00 results in 5, indicating that Roy bought five whole pizzas.

Roy spent a total of P 1070 for pizza with 3 sets of additional toppings. Since each set of additional toppings costs P 40.00, then the total cost of the toppings is 3 x P 40.00 = P 120.00. Subtracting this from the total amount spent gives us P 950.00, which is the cost of the pizzas alone.

Since each whole pizza costs P 190.00, we can divide the cost of the pizzas by the cost of each pizza to find the number of whole pizzas Roy bought. Therefore, P 950.00 / P 190.00 = 5.

Thus, Roy bought 5 whole pizzas.

Learn more about arithmetic here: brainly.com/question/11559160

#SPJ4

An unfair coin with Pr[H]=23 is flipped. If the flip results in a head, then a marble is selected from an urn containing 6 red, 9 white, and 10 blue marbles. If the flip results in a tail then a marble is selected from an urn containing 10 red and 1 white marbles. If the marble selected is white, then what is the probability that a flip resulted in a head?

Answers

The probability that the flip results in a head is given as Pr[H] = 23. Therefore, the probability that the flip results in a tail is Pr[T] = 1 - Pr[H] = 1 - 23 = 13.

Let A be the event that a white marble is selected. We need to find the conditional probability Pr[H|A], i.e., the probability that the flip resulted in a head given that a white marble was selected.

Using Bayes' theorem, we have:

Pr[H|A] = (Pr[A|H]*Pr[H]) / Pr[A]

Pr[A|H] is the probability of selecting a white marble given that the flip resulted in a head. This is given by (9/25), since there are 9 white marbles out of 25 in the first urn.

Pr[A] is the total probability of selecting a white marble, which can be found using the law of total probability:

Pr[A] = Pr[A|H]*Pr[H] + Pr[A|T]*Pr[T]

= (9/25)*0.23 + (1/11)*0.13

= 0.0888 + 0.0118

= 0.1006

Pr[A|T] is the probability of selecting a white marble given that the flip resulted in a tail. This is given by (1/11), since there is only 1 white marble out of 11 in the second urn.

Therefore,

Pr[H|A] = (9/25 * 0.23) / 0.1006 = 0.6508

Hence, the probability that the flip resulted in a head given that a white marble was selected is 0.6508 (or approximately 0.65).

For more questions like Probability click the link below:

https://brainly.com/question/30034780

#SPJ11

What is the area of the​ triangle?

Answers

Answer:

Step-by-step explanation:

Given:

Side a and Side b are 6 and 5.

The angle C is 131.

This is an obtuse scalene triangle as identified.

Area = ab * sin (C)/2  = 11.32064

PLEASE HELP!!
20 POINTS


A rental car company offers two rental plans, Plan A and Plan B, for the same economy size car. For both plans, the total rental cost f(m)
is a function of the number of miles m
that the car is driven.

Plan A: f(m)= 0.12+75


Plan B: f(m)+0.35

I. In complete sentences, translate each function into a verbal model describing the total cost of the rental in terms of the number of miles that the car is driven.

II. For each function, determine how the rate of change will affect the total cost of a car rental.

III. For a car rental that will include a maximum of 250 miles for the duration of the rental, which plan is the most cost effective?

Answers

Answer: At 250 miles, plan B is the most cost effective.

Step-by-step explanation:

Answers are under the questions below.

I. In complete sentences, translate each function into a verbal model describing the total cost of the rental in terms of the number of miles that the car is driven.

Plan A will charge .12 per mile plus a one time fee of $75.

Equation is y = .12x + 75 with the $75 being the y intercept or one time fee.

Plan B will charge .35 per mile with no one time fee.

Equation is y = .35x with the y intercept being (0) zero.

II. For each function, determine how the rate of change will affect the total cost of a car rental.

Initially the Plan A will be more expensive because of the one time fee, even though the rate is .12 per mile, which is much less than Plan B.

Plan B charges .35 per mile, but will eventually catch up in cost.

At approximately 326 miles the cost of each plan will equal at $114.13.

After 326 miles, Plan B will cost more than Plan A.

III. For a car rental that will include a maximum of 250 miles for the duration of the rental, which plan is the most cost effective?

Plan A, substitute 250 miles for x

Cost = .12 (250)  + 75

Cost = $105

Plan B, substitute 250 miles for x

Cost = .35 (250)

Cost = $87.50

At 250 miles, plan B is the most cost effective, saving $17.50.

graph is attached.

DNA on the Ocean Floor (adapted from Baldi book and cont'd from homework 4)- DNA occurs on the ocean floor (outside of living cells) where it plays an important role in nourishing seafloor life. A random sample of ocean floor specimens from 116 locations around the world gives mean sample DNA concentration Xbar=0.2781g/m2 and sample standard deviation s=0.1803g/m2. A healthy concentration of ocean floor DNA is considered to be around 0.31 g/m2.
a. Use the p-value approach to test if the floor specimens mean DNA concentration are different to the what is considered a healthy concentration. Use alpha = 0.05. Start by writing the null and alternative hypothesis. Make sure you write a conclusion regarding the question about the floor specimen's DNA concentration. (1pt)
b. What if the question was: test if the floor specimens mean DNA concentration were less than what is considered a healthy concentration? What would the p- value be? (0.5 pts)
c. Repeat the one-sided test from b. using the 95% confidence interval approach. What do you conclude?

Answers

All parts are define in the below points.

Define the term random sample?

A random sample is a subset of a population in which each individual or element in the population has an equal chance of being selected. It is a sampling method used in statistics and research to minimize bias and increase the generalizability of the findings to the larger population.

a. Hypotheses: Null Hypothesis: The mean DNA concentration of the ocean floor specimens is not significantly different from the healthy concentration (µ = 0.31g/m2). Alternative Hypothesis: The mean DNA concentration of the ocean floor specimens is significantly different from the healthy concentration (µ ≠ 0.31g/m2). Using a two-tailed t-test with alpha = 0.05, we find a p-value of 0.0003, which is less than the significance level. Therefore, we reject the null hypothesis and conclude that the mean DNA concentration of the ocean floor specimens is significantly different from the healthy concentration.

b. We would perform a one-tailed t-test with the alternative hypothesis that the mean DNA concentration is less than 0.31g/m2 if the goal was to determine whether the mean DNA concentration of the floor specimens was lower than what is regarded as a healthy concentration. It would have a p-value of 0.00015.

c. Using the 95% confidence interval approach, we construct a one-sided confidence interval for the mean DNA concentration. If the lower bound of the confidence interval is less than 0.31g/m2, we can conclude that the mean DNA concentration is less than the healthy concentration. The 95% confidence interval for the mean is (0.2457g/m2, 0.3105g/m2), which does not include the healthy concentration of 0.31g/m2. Therefore, we can conclude that the mean DNA concentration of the ocean floor specimens is less than the healthy concentration.

To know more about random samples, visit:

https://brainly.com/question/24466382

#SPJ1

a). We discover a p-value of 0.0003 using a two-tailed t-test with alpha = 0.05, which is below the significance level.

b). Its p-value would be 0.00015.

c). The safe concentration of [tex]0.31g/m^2[/tex] is not included in the 95% confidence interval for the mean, which is [tex](0.2457g/m^2,\ 0.3105g/m^2)[/tex].

Define the term random sample?

A random sample is a portion of a community in which every person or component has an equal chance of being chosen. In statistics and research, it is a sampling technique used to reduce bias and improve the generalizability of the results to a broader population.

A). An hypothesis is a The null hypothesis states that there is no discernible difference between the mean DNA concentration of the ocean bottom samples and the healthy concentration [tex](\mu=0.31g/m^2)[/tex]. Alternative Hypothesis: The mean DNA concentration of the ocean floor samples differs considerably from the healthy concentration [tex](\mu\neq 0.31g/m^2)[/tex] in a statistically significant way. We discover a p-value of 0.0003 using a two-tailed t-test with alpha = 0.05, which is below the significance level. We therefore reject the null hypothesis and come to the conclusion that the mean DNA concentration of the samples from the ocean bottom differs significantly from that of healthy individuals.

B). If the objective was to determine whether the mean DNA concentration of the floor specimens was lower than what is considered as a healthy concentration, we would conduct a one-tailed t-test with the alternative hypothesis that the mean DNA concentration is less than [tex]0.31g/m^2[/tex]. Its p-value would be 0.00015.

C). We create a one-sided confidence interval for the mean DNA concentration using the 95% confidence interval method. The mean DNA concentrationis less than the healthy concentration if the lower limit of the confidence interval is less than [tex]0.31g/m^2[/tex]. The safe concentration of [tex]0.31g/m^2[/tex] is not included in the 95% confidence interval for the mean, which is [tex](0.2457g/m^2,\ 0.3105g/m^2)[/tex]. As a result, we can say that the average DNA concentration of the samples from the ocean bottom is lower than the healthy concentration.

To know more about random sample, visit:

brainly.com/question/24466382

#SPJ1

HELP ASAP PLEASE!! A yard plan includes a rectangular garden that is surrounded by bricks. In the drawing, the garden is 7 inches by 4 inches. The length and width of the actual garden will be 35 times larger than the length and width in the drawing.
What is the perimeter of the drawing? Show your work.
What is the perimeter of the actual garden? Show your work.
What is the effect on the perimeter of the garden with the dimensions are multiplied by 35? Show your work.

Answers

Answer:

(a) perimeter of the drawing = (7 x2) + (4 x 2)

                                               = 22 inches

(b) perimeter of the actual garden = 2(7 x 35) + 2(4 x 35)

                                                         = 490 + 280

                                                         = 770 inches

(c) 22 x 35 = 770

i don't really understand the last question. sorry if i gave you the wrong answers

A student takes a multiple-choice test that has 10 questions. Each question has four choices. The student guesses randomly at each answer. Round the answers to three decimal places Part 1 of2 (a) Find P(5) P(5)- Part 2 of2 (b) Find P(More than 3) P(More than 3)

Answers

A student attempts a 10-question multiple-choice test where each question presents four options, and the student makes random guesses for each answer. So the probability of (a) P(5)= 0.058 and (b) P(More than 3)= 0.093.

Part 1: Calculation of probability of getting 5 questions correct

(a) P(5)The formula used to find the probability of getting a certain number of questions correct is:

P(k) = (nCk)pk(q(n−k))

Where, n = total number of questions

(10)k = number of questions that are answered correctly

p = probability of getting any question right = 1/4

q = probability of getting any question wrong = 3/4

P(5) = P(k = 5) = (10C5)(1/4)5(3/4)5= 252 × 0.0009765625 × 0.2373046875≈ 0.058

Part 2: Calculation of probability of getting more than 3 questions correct

(b) P(More than 3) = P(k > 3) = P(k = 4) + P(k = 5) + P(k = 6) + P(k = 7) + P(k = 8) + P(k = 9) + P(k = 10)

P(k = 4) = [tex]10\choose4[/tex](1/4)4(3/4)6 = 210 × 0.00390625 × 0.31640625 ≈ 0.02

P(k = 5) = [tex]10\choose5[/tex](1/4)5(3/4)5 = 252 × 0.0009765625 × 0.2373046875 ≈ 0.058

P(k = 6) = [tex]10\choose6[/tex](1/4)6(3/4)4 = 210 × 0.0002441406 × 0.31640625 ≈ 0.012

P(k = 7) = [tex]10\choose7[/tex](1/4)7(3/4)3 = 120 × 0.00006103516 × 0.421875 ≈ 0.002

P(k = 8) = [tex]10\choose8[/tex](1/4)8(3/4)2 = 45 × 0.00001525878 × 0.5625 ≈ 0.001

P(k = 9) = [tex]10\choose9[/tex](1/4)9(3/4)1 = 10 × 0.000003814697 × 0.75 ≈ 0.000

P(k = 10) = [tex]10\choose10[/tex](1/4)10(3/4)0 = 1 × 0.0000009536743 × 1 ≈ 0

P(More than 3) = 0.020 + 0.058 + 0.012 + 0.002 + 0.001 + 0.000 + 0≈ 0.093

Therefore, the probabilities of the given situations are: P(5) ≈ 0.058, P(More than 3) ≈ 0.093.

To know more about probability: https://brainly.com/question/251701

#SPJ11

A fruit basket holds x2 x2 – 3 x x + 12 apples. Maha takes out 4 x− x− 6 of them. How many apples are left in the basket?
p.s Please explain how you solve this question!

Answers

The answer of the given question based on the given equation  that fruit basket holds x2 x2 – 3 x x + 12 apples and Maha takes out 4 x− x− 6 of them the answer is , there are x² - 6x + 18 apples left in the basket after Maha takes out 4x - x - 6 of them.

What is Expression?

In mathematics, expression is  combination of numbers, variables, and mathematical operations that can be evaluated to produce  value. An expression can be as simple as  single number or variable, or it can be complex, involving many different operations and variables.

Expressions can be used to represent many different mathematical ideas, like equations, inequalities, functions, and more. They can be used to model real-world situations, make predictions, and solve problems in wide variety of fields, like physics, economics, engineering, and more.

The fruit basket initially holds x² - 3x + 12 apples. If Maha takes out 4x - x - 6 of them, then we can subtract this expression from the initial number of apples to find how many are left in the basket.

So, the expression for the number of apples left in the basket is:

x² - 3x + 12 - (4x - x - 6)

We can simplify this by combining like terms:

x² - 3x + 12 - 4x + x + 6

Simplifying further:

x² - 6x + 18

Therefore, there are x² - 6x + 18 apples left in the basket after Maha takes out 4x - x - 6 of them.

It's important to note that we cannot simplify 4x - x - 6 to 3x - 6 in this case because the two terms have different coefficients. Instead, we need to distribute the negative sign to both terms inside the parentheses to get 4x - x - 6 = 4x - 1x + (-6) = 3x - 6, which we can then use in the expression for the number of apples left in the basket.

To know more about Variable visit:

https://brainly.com/question/2466865

#SPJ1

A square is inscribed in a right triangle with leg lengths 6 and 8 so that they have a common right angle. FInd the square's side length.

Answers

Answer:

10 units

Step-by-step explanation:

Here, legs = base and perpendiculars.

So, Clearly given Base = 6 units Perpendicular = 8 cm

Square's Side = Hypotenuse.

By Pythagoras theorem,

H² = B²+P²

H ² = 6²+8²

H² = 36+64 = (10)²

H = 10 units.

Square's Side length = 10 units

Thabang save money by putting coins in a money box. The money box has 600 coins that consist of 20 cents and 50 cents
So calculate how many 50 cents pieces are in the container if there are 220 pieces of 20 cents

Answers

The number of 50 cents in the container is 380 fifty cents

How to find the number of 50 cents in the container?

Since Thabang save money by putting coins in a money box. The money box has 600 coins that consist of 20 cents and 50 cents

To calculate how many 50 cents pieces are in the container if there are 220 pieces of 20 cents, we proceed as follows.

Let

x = number of 20 cents and y = number of 50 cents

Since the total number of cents in the container is 600, we have that

x + y = 600

So, making y subject of the formula, we have that

y = 600 - x

Since x = 220

y = 600 - 220

= 380

So, there are 380 fifty cents

Learn more about number of cents in container here:

https://brainly.com/question/23370723

#SPJ1

What is the value of x in the equation 1/4(4 + x) = 4/3

Answers

The value of x in the equation 1/4(4 + x) = 4/3 is x = 4/3.

Multiply both sides of the equation by 4 to eliminate the fraction on the left-hand side:

1/4(4 + x) = 4/3

4 * 1/4(4 + x) = 4 * 4/3

Simplifying:

4 + x = 16/3

Subtract 4 from both sides of the equation:

4 + x - 4 = 16/3 - 4

Simplifying:

x = 16/3 - 12/3

x = 4/3

A fraction is a mathematical concept used to represent a part of a whole or a ratio between two quantities. It is typically written in the form of a numerator (top number) over a denominator (bottom number), separated by a horizontal line. For example, the fraction 1/2 represents one out of two equal parts, or half of a whole. Similarly, the fraction 3/4 represents three out of four equal parts, or three-quarters of a whole.

Fractions are an essential part of mathematics and are used in a wide range of applications, including measurements, cooking, and financial calculations. They can be added, subtracted, multiplied, and divided just like whole numbers, but they require a bit more care in their manipulation due to their unique structure.

To learn more about Fraction visit here:

brainly.com/question/10354322

#SPJ4

Other Questions
which of the following is not a significant factor contributing to the damage caused by an earthquake? 3. Look at the graph's 1959 totals. What does the 4.30 level indicate?What does the 4.60 indicate?What does the 1.66 indicate? Name a type of fuel that is a solid? TRUE OR FALSE to calculate the average of the numeric values in a list, the first step is to get the total of values in the list. HELP! I WILL MAKE U BRAINLIST! DUE BY 12:00 AM! I have inserted what you need to do and just search Buzzfeed "Can TV Make Us Not Hate Ourselves?" and read it! An entrepreneur decided to leave a job that pays $50,000 a year to start a business. These lost wages would be considered ______________ .Select the correct answer below:A. economic profitB. an explicit costC. an implicit costD. accounting profit What is the electromagnetic force?A. a force that governs how elements break down naturallyB. a force that holds atomic nuclei togetherC. a force that attracts objects with mass towards each otherD. a force that acts on charged particles What will be displayed after this code segment is run?gems -"ruby", "sapphire", "garnet""opal", "emerald"gemsDISPLAY gems 2-"Opal"-" sapphire"-" granite"-"Emerald" Why is the Arc de triumph like the colosseum? In many cases, animals display structural traits of their ancestors only during embryonic development. Which of the following aquatic organisms might display limbs during embryonic development?blue whaleskiller whales A well-connected person who may not make the buying decision but who may have an impact on the person who does is a:A) gatekeeperB) referralC) prospectD) target marketE) center of influence It gives me pleasure to announce to Congress that benevolent policy of the Government,steadily pursued for nearly thirty years,in relation to the removal of the Indians beyond the white settlements is approaching to a happy consummation. Two important tribes have the provision made for their removal the last session of Congress,and it is believed that their example induce the remaining to seek the same obvious advantages. What tone does this passage most convey? A. Disappointed B. CasualC. AngryD. Professional Is the function represented by the following table linear, quadratic or exponential? the right of every citizen against arbitrary action by national or state governments is known as what? The frequency with which margin accounts are adjusted for gains and losses is:?QuarterlyDailyMonthlyWeekly 5. A continuous hot rolling mill has two stands. Thickness of the starting plate = 1.0 in. and width = 12.0 in. Final thickness is to be 0.5 in. Roll radius at each stand = 10 in. Rotational speed at the first stand = 20 rev/min. Equal drafts of 0.25 in. are to be taken at each stand. The plate is wide enough relative to its thickness that no increase in width occurs. Under the assumption that the forward slip is equal at each stand determine (a) speed v, at each stand and (b) forward slip s. (c) also determine the exiting speeds at each rolling stand if the entering speed at the first stand is 85 ft/min. Which of the following terms refers to a passage or channel and is derived from the Latin meaning passage? -Concha - Naris - Turbinate - Sinus - Meatus. Mean arterial pressure within the circulatory system is constantly monitored by: a. endothelial cellsb. heart sensorsc. baroreceptorsd. ganglionse. pressure sinuses a new project has an initial cost of $165,000. the equipment will be depreciated on a straight-line basis to a zero book value over the five-year life of the project. the projected net income each year is $12,400, $16,300, $18,200, $14,300, and $10,200, respectively. what is the average accounting return? the coefficients used are larger than necessary. Rewrite this balanced equation using the smallest coefficients possible.9Fe(s) + 12H20(I) 3Fe3O4(s) + 12H2(9)